Search found 788 matches


Re: DS GMATPrep

1 - insufficient.
Take {a = 8, b = 4, x = -0.5}, {a = 8, b = -4, x = 0.5}
x can be positive or negative

2 - insufficient
Take {a = 8, b = 4, x = -0.5}, {a = 8, b = -4, x = 0.5}

Together, same story.

Hence E

by jayhawk2001

Fri Dec 14, 2007 11:35 am
Forum: Data Sufficiency
Topic: DS GMATPrep
Replies: 2
Views: 1757

Re: pls help

I think the question is asking for a weighted average

i.e. .2 * 800k + .5 * 500k + .3*350k

by jayhawk2001

Fri Dec 14, 2007 11:17 am
Forum: Problem Solving
Topic: pls help
Replies: 4
Views: 1950

Re: How would you interpret this question?

Riggz wrote:I cant remember the wording exactly.

X is equal to Y plus 40%. Z is equal to Y less 20%.

X = y + 40% or x = 1.40y

Z= y - 20% or z = 0.80y
One tends to say % of something. Percent by itself does not have
any meaning. Given the above, I'm inclined towayds 1.4y and .8y

by jayhawk2001

Fri Dec 14, 2007 11:06 am
Forum: Problem Solving
Topic: How would you interpret this question?
Replies: 2
Views: 1663

Re: Geometry problem

If the area of square S and the area of circle C are equal, then the ratio of the perimeter of S to the circumference of C is closest to ... a)7/9 b)8/9 c)9/8 d)4/3 e)2/1 Not sure how to do this. s^2 = pi * r^2 s/r = sqrt(pi) So, 4s / (2*pi*r) = 2 / sqrt(pi) sqrt(pi) = sqrt(3.14) ~ 1.7 So, we know ...

by jayhawk2001

Wed Dec 12, 2007 6:29 pm
Forum: Problem Solving
Topic: Geometry problem
Replies: 5
Views: 1816

xcise_science wrote:Hi,
I don't understand how you concluded that its a right triangle thats formed.
Please check

http://www.beatthegmat.com/ps-t6611.html

by jayhawk2001

Wed Dec 12, 2007 10:01 am
Forum: Problem Solving
Topic: 2 problems from gmatprep
Replies: 3
Views: 1892

1 - insufficient. m/y = x/r. We don't know if m/r = x/y

2 - sufficient. my + xy = xr + xy.
So, m/r = x/y

Hence B

by jayhawk2001

Tue Dec 11, 2007 5:47 pm
Forum: Data Sufficiency
Topic: DS - ratio/propotion : confused.
Replies: 1
Views: 1528

StarDust845 wrote:Why is the answer D?

I see it this way.

From stmt (1) we get k^2 - k - 1 = 0

If you solve this, you get k = (1 + Sqrt(5))/2 or k = (1 - sqrt(5))/2
So the answer can't be A or D.
We don't have to solve for k here. Question asks for k^2 - k.

by jayhawk2001

Tue Dec 11, 2007 5:40 pm
Forum: Data Sufficiency
Topic: DS
Replies: 3
Views: 1885

Re: absolute

magical cook wrote:if y>=0, what is the value of x?

1) lx-3l >= y

2) lx-3l <y>= 0.
Since y >=0, only value of y that satisfies this is y=0.

So |x-3| <= 0. So, we can find x.

Hence B

by jayhawk2001

Tue Dec 11, 2007 11:51 am
Forum: Data Sufficiency
Topic: absolute
Replies: 10
Views: 2463

Only E seems to be finite and bounded.

Can you please post option C. I don't see a choice 'c' in the
question

by jayhawk2001

Sun Dec 09, 2007 8:37 am
Forum: Problem Solving
Topic: inequity
Replies: 6
Views: 1902

Re: Integers

[quote="salma"]Is n an integer? (1) n^2 is an integer (2) square root of n is an integer The answers says that from (2) we can know that n is an integer but not from (1). I cannot think of a non integer which square root is an integer? Thx[/quote] 1 - insufficient. Knowing n^2 is an intege...

by jayhawk2001

Sat Dec 08, 2007 4:38 pm
Forum: Data Sufficiency
Topic: Integers
Replies: 2
Views: 1733

oops sorry, I attached the wrong image. here's the right one. Is it possible to delete the old one? Thanks I was wondering where GCF and LCM are in the previous question :-) Product of 2 numbers = LCM * GCF [ For reference: http://en.wikipedia.org/wiki/Greatest_common_divisor#Calculating_the_gcd ] ...

by jayhawk2001

Sat Dec 08, 2007 4:35 pm
Forum: Data Sufficiency
Topic: LCM & GSF DS probelm
Replies: 3
Views: 1917

1 - insufficient. take d=2, d=3. 100 and 1000 are factors of 30!

2 - insufficient. again, d could be anything

30! has 7 zeroes (5*2, 10, 15*4, 20, 25*8, 30 all yield zeroes). So,
knowing that d>6 is sufficient

Is it C

by jayhawk2001

Sat Dec 08, 2007 10:13 am
Forum: Data Sufficiency
Topic: LCM & GSF DS probelm
Replies: 3
Views: 1917

Let eqn of line n: y = m1x + c1
Eqn of line p: y = m2x + c2

1 - insufficient. we just have
1 = 5m1 + c1
1 = 5m2 + c2

2 - insufficient.
we just know c1 > c2

combining the two we can see that m2 > m1. Hence sufficient.

by jayhawk2001

Sat Dec 08, 2007 9:51 am
Forum: Data Sufficiency
Topic: DS Coordinate Geometry
Replies: 3
Views: 2360

3C2 * 3C2 * 2C1 / 8C5
= 9/28

Is it B?

by jayhawk2001

Sat Dec 08, 2007 9:42 am
Forum: Problem Solving
Topic: PS, tough probability
Replies: 14
Views: 5051

I get 1/32 as well

We are asked to find probability of HHHTT from the 32 possible
combinations. So, 1/32

by jayhawk2001

Sat Dec 08, 2007 9:37 am
Forum: Problem Solving
Topic: coin flipped?
Replies: 6
Views: 1915